exercise:F4bf733f21: Difference between revisions

From Stochiki
(Created page with "<div class="d-none"><math> \newcommand{\NA}{{\rm NA}} \newcommand{\mat}[1]{{\bf#1}} \newcommand{\exref}[1]{\ref{##1}} \newcommand{\secstoprocess}{\all} \newcommand{\NA}{{\rm NA}} \newcommand{\mathds}{\mathbb}</math></div> Assume that the probability of a “success” on a single experiment with <math>n</math> outcomes is <math>1/n</math>. Let <math>m</math> be the number of experiments necessary to make it a favorable bet that at least one success will occur (see...")
 
No edit summary
 
Line 1: Line 1:
<div class="d-none"><math>
Assume that the probability of a “success” on a single experiment with <math>n</math> outcomes is <math>1/n</math>.  Let <math>m</math> be the number of experiments necessary to make it a favorable bet that at least one success
\newcommand{\NA}{{\rm NA}}
will occur (see [[exercise:9e83c3ee89 |Exercise]]).
\newcommand{\mat}[1]{{\bf#1}}
 
\newcommand{\exref}[1]{\ref{##1}}
<ul style="list-style-type:lower-alpha"><li> Show that the probability that, in <math>m</math> trials, there are no successes
\newcommand{\secstoprocess}{\all}
\newcommand{\NA}{{\rm NA}}
\newcommand{\mathds}{\mathbb}</math></div>
Assume that the probability of a “success” on
a single experiment with <math>n</math> outcomes is <math>1/n</math>.  Let <math>m</math> be the number of
experiments necessary to make it a favorable bet that at least one success
will
occur (see [[guide:4f3a4e96c3#sec 1.1 [[guide:4f3a4e96c3#exer 1.1.5 ||Exercise]].]]).
<ul><li> Show that the probability that, in <math>m</math> trials, there are no successes
is <math>(1 - 1/n)^m</math>.
is <math>(1 - 1/n)^m</math>.
</li>
</li>

Latest revision as of 22:06, 12 June 2024

Assume that the probability of a “success” on a single experiment with [math]n[/math] outcomes is [math]1/n[/math]. Let [math]m[/math] be the number of experiments necessary to make it a favorable bet that at least one success will occur (see Exercise).

  • Show that the probability that, in [math]m[/math] trials, there are no successes is [math](1 - 1/n)^m[/math].
  • (de Moivre) Show that if [math]m = n \log 2[/math] then
    [[math]] \lim_{n \to \infty} \left(1 - \frac1n \right)^m = \frac12\ . [[/math]]
    Hint:
    [[math]] \lim_{n \to \infty} \left(1 - \frac1n \right)^n = e^{-1}\ . [[/math]]
    Hence for large [math]n[/math] we should choose [math]m[/math] to be about [math]n \log 2[/math].
  • Would DeMoivre have been led to the correct answer for de Méré's two bets if he had used his approximation?